GET READY FOR THE AMC 12 WITH AoPS
Learn with outstanding instructors and top-scoring students from around the world in our AMC 12 Problem Series online course.
CHECK SCHEDULE

Difference between revisions of "2021 AMC 12B Problems"

(Created page with "{{AMC12 Problems|year=2021|ab=B}} ==Problem 1== These problems will not be posted until the 2021 AMC 12B is released on Thursday, February 10, 2021. 2021 AMC 12B Problems/P...")
 
 
(56 intermediate revisions by 16 users not shown)
Line 1: Line 1:
 
{{AMC12 Problems|year=2021|ab=B}}
 
{{AMC12 Problems|year=2021|ab=B}}
 
==Problem 1==
 
==Problem 1==
These problems will not be posted until the 2021 AMC 12B is released on Thursday, February 10, 2021.
+
How many integer values of <math>x</math> satisfy <math>|x|<3\pi?</math>
 +
 
 +
<math>\textbf{(A) }9 \qquad \textbf{(B) }10 \qquad \textbf{(C) }18 \qquad \textbf{(D) }19 \qquad \textbf{(E) }20</math>
  
 
[[2021 AMC 12B Problems/Problem 1|Solution]]
 
[[2021 AMC 12B Problems/Problem 1|Solution]]
  
 
==Problem 2==
 
==Problem 2==
These problems will not be posted until the 2021 AMC 12B is released on Thursday, February 10, 2021.
+
At a math contest, <math>57</math> students are wearing blue shirts, and another <math>75</math> students are wearing yellow shirts. The <math>132</math> students are assigned into <math>66</math> pairs. In exactly <math>23</math> of these pairs, both students are wearing blue shirts. In how many pairs are both students wearing yellow shirts?
 +
 
 +
<math>\textbf{(A) }23 \qquad \textbf{(B) }32 \qquad \textbf{(C) }37 \qquad \textbf{(D) }41 \qquad \textbf{(E) }64</math>
  
 
[[2021 AMC 12B Problems/Problem 2|Solution]]
 
[[2021 AMC 12B Problems/Problem 2|Solution]]
  
 
==Problem 3==
 
==Problem 3==
These problems will not be posted until the 2021 AMC 12B is released on Thursday, February 10, 2021.
+
Suppose<cmath>2+\frac{1}{1+\frac{1}{2+\frac{2}{3+x}}}=\frac{144}{53}.</cmath>What is the value of <math>x?</math>
 +
 
 +
<math>\textbf{(A) }\frac34 \qquad \textbf{(B) }\frac78 \qquad \textbf{(C) }\frac{14}{15} \qquad \textbf{(D) }\frac{37}{38} \qquad \textbf{(E) }\frac{52}{53}</math>
  
 
[[2021 AMC 12B Problems/Problem 3|Solution]]
 
[[2021 AMC 12B Problems/Problem 3|Solution]]
  
 
==Problem 4==
 
==Problem 4==
These problems will not be posted until the 2021 AMC 12B is released on Thursday, February 10, 2021.
+
Ms. Blackwell gives an exam to two classes. The mean of the scores of the students in the morning class is <math>84</math>, and the afternoon class's mean score is <math>70</math>. The ratio of the number of students in the morning class to the number of students in the afternoon class is <math>\frac34</math>. What is the mean of the score of all the students?
 +
 
 +
<math>\textbf{(A) }74 \qquad \textbf{(B) }75 \qquad \textbf{(C) }76 \qquad \textbf{(D) }77 \qquad \textbf{(E) }78</math>
  
 
[[2021 AMC 12B Problems/Problem 4|Solution]]
 
[[2021 AMC 12B Problems/Problem 4|Solution]]
  
 
==Problem 5==
 
==Problem 5==
These problems will not be posted until the 2021 AMC 12B is released on Thursday, February 10, 2021.
+
The point <math>P(a,b)</math> in the <math>xy</math>-plane is first rotated counterclockwise by <math>90^\circ</math> around the point <math>(1,5)</math> and then reflected about the line <math>y=-x</math>. The image of <math>P</math> after these two transformations is at <math>(-6,3)</math>. What is <math>b-a?</math>
 +
 
 +
<math>\textbf{(A) }1 \qquad \textbf{(B) }3 \qquad \textbf{(C) }5 \qquad \textbf{(D) }7 \qquad \textbf{(E) }9</math>
  
 
[[2021 AMC 12B Problems/Problem 5|Solution]]
 
[[2021 AMC 12B Problems/Problem 5|Solution]]
  
 
==Problem 6==
 
==Problem 6==
These problems will not be posted until the 2021 AMC 12B is released on Thursday, February 10, 2021.
+
An inverted cone with base radius <math>12 \text{cm}</math> and height <math>18\text{cm}</math> is full of water. The water is poured into a tall cylinder whose horizontal base has a radius of <math>24\text{cm}</math>. What is the height in centimeters of the water in the cylinder?
 +
 
 +
<math>\textbf{(A) }1.5 \qquad \textbf{(B) }3 \qquad \textbf{(C) }4 \qquad \textbf{(D) }4.5 \qquad \textbf{(E) }6</math>
  
 
[[2021 AMC 12B Problems/Problem 6|Solution]]
 
[[2021 AMC 12B Problems/Problem 6|Solution]]
  
 
==Problem 7==
 
==Problem 7==
These problems will not be posted until the 2021 AMC 12B is released on Thursday, February 10, 2021.
+
Let <math>N=34\cdot34\cdot63\cdot270.</math> What is the ratio of the sum of the odd divisors of <math>N</math> to the sum of the even divisors of <math>N?</math>
 +
 
 +
<math>\textbf{(A) }1:16 \qquad \textbf{(B) }1:15 \qquad \textbf{(C) }1:14 \qquad \textbf{(D) }1:8 \qquad \textbf{(E) }1:3</math>
  
 
[[2021 AMC 12B Problems/Problem 7|Solution]]
 
[[2021 AMC 12B Problems/Problem 7|Solution]]
  
 
==Problem 8==
 
==Problem 8==
These problems will not be posted until the 2021 AMC 12B is released on Thursday, February 10, 2021.
+
Three equally spaced parallel lines intersect a circle, creating three chords of lengths <math>38,38,</math> and <math>34</math>. What is the distance between two adjacent parallel lines?
 +
 
 +
<math>\textbf{(A) }5\frac12 \qquad \textbf{(B) }6 \qquad \textbf{(C) }6\frac12 \qquad \textbf{(D) }7 \qquad \textbf{(E) }7\frac12</math>
  
 
[[2021 AMC 12B Problems/Problem 8|Solution]]
 
[[2021 AMC 12B Problems/Problem 8|Solution]]
  
 
==Problem 9==
 
==Problem 9==
These problems will not be posted until the 2021 AMC 12B is released on Thursday, February 10, 2021.
+
What is the value of<cmath>\frac{\log_2 80}{\log_{40}2}-\frac{\log_2 160}{\log_{20}2}?</cmath>
 +
<math>\textbf{(A) }0 \qquad \textbf{(B) }1 \qquad \textbf{(C) }\frac54 \qquad \textbf{(D) }2 \qquad \textbf{(E) }\log_2 5</math>
  
 
[[2021 AMC 12B Problems/Problem 9|Solution]]
 
[[2021 AMC 12B Problems/Problem 9|Solution]]
  
 
==Problem 10==
 
==Problem 10==
These problems will not be posted until the 2021 AMC 12B is released on Thursday, February 10, 2021.
+
Two distinct numbers are selected from the set <math>\{1,2,3,4,\dots,36,37\}</math> so that the sum of the remaining <math>35</math> numbers is the product of these two numbers. What is the difference of these two numbers?
 +
 
 +
<math>\textbf{(A) }5 \qquad \textbf{(B) }7 \qquad \textbf{(C) }8\qquad \textbf{(D) }9 \qquad \textbf{(E) }10</math>
  
 
[[2021 AMC 12B Problems/Problem 10|Solution]]
 
[[2021 AMC 12B Problems/Problem 10|Solution]]
  
 
==Problem 11==
 
==Problem 11==
These problems will not be posted until the 2021 AMC 12B is released on Thursday, February 10, 2021.
+
Triangle <math>ABC</math> has <math>AB=13,BC=14</math> and <math>AC=15</math>. Let <math>P</math> be the point on <math>\overline{AC}</math> such that <math>PC=10</math>. There are exactly two points <math>D</math> and <math>E</math> on line <math>BP</math> such that quadrilaterals <math>ABCD</math> and <math>ABCE</math> are trapezoids. What is the distance <math>DE?</math>
 +
 
 +
<math>\textbf{(A) }\frac{42}5 \qquad \textbf{(B) }6\sqrt2 \qquad \textbf{(C) }\frac{84}5\qquad \textbf{(D) }12\sqrt2 \qquad \textbf{(E) }18</math>
  
 
[[2021 AMC 12B Problems/Problem 11|Solution]]
 
[[2021 AMC 12B Problems/Problem 11|Solution]]
  
 
==Problem 12==
 
==Problem 12==
These problems will not be posted until the 2021 AMC 12B is released on Thursday, February 10, 2021.
+
Suppose that <math>S</math> is a finite set of positive integers. If the greatest integer in <math>S</math> is removed from <math>S</math>, then the average value (arithmetic mean) of the integers remaining is <math>32</math>. If the least integer in <math>S</math> is also removed, then the average value of the integers remaining is <math>35</math>. If the greatest integer is then returned to the set, the average value of the integers rises to <math>40</math>. The greatest integer in the original set <math>S</math> is <math>72</math> greater than the least integer in <math>S</math>. What is the average value of all the integers in the set <math>S</math>?
 +
 
 +
<math>\textbf{(A) }36.2 \qquad \textbf{(B) }36.4 \qquad \textbf{(C) }36.6\qquad \textbf{(D) }36.8 \qquad \textbf{(E) }37</math>
  
 
[[2021 AMC 12B Problems/Problem 12|Solution]]
 
[[2021 AMC 12B Problems/Problem 12|Solution]]
  
 
==Problem 13==
 
==Problem 13==
These problems will not be posted until the 2021 AMC 12B is released on Thursday, February 10, 2021.
+
How many values of <math>\theta</math> in the interval <math>0<\theta\le 2\pi</math> satisfy<cmath>1-3\sin\theta+5\cos3\theta = 0?</cmath>
 +
<math>\textbf{(A) }2 \qquad \textbf{(B) }4 \qquad \textbf{(C) }5\qquad \textbf{(D) }6 \qquad \textbf{(E) }8</math>
  
 
[[2021 AMC 12B Problems/Problem 13|Solution]]
 
[[2021 AMC 12B Problems/Problem 13|Solution]]
  
 
==Problem 14==
 
==Problem 14==
These problems will not be posted until the 2021 AMC 12B is released on Thursday, February 10, 2021.
+
Let <math>ABCD</math> be a rectangle and let <math>\overline{DM}</math> be a segment perpendicular to the plane of <math>ABCD</math>. Suppose that <math>\overline{DM}</math> has integer length, and the lengths of <math>\overline{MA},\overline{MC},</math> and <math>\overline{MB}</math> are consecutive odd positive integers (in this order). What is the volume of pyramid <math>MABCD?</math>
 +
 
 +
<math>\textbf{(A) }24\sqrt5 \qquad \textbf{(B) }60 \qquad \textbf{(C) }28\sqrt5\qquad \textbf{(D) }66 \qquad \textbf{(E) }8\sqrt{70}</math>
  
 
[[2021 AMC 12B Problems/Problem 14|Solution]]
 
[[2021 AMC 12B Problems/Problem 14|Solution]]
  
 
==Problem 15==
 
==Problem 15==
These problems will not be posted until the 2021 AMC 12B is released on Thursday, February 10, 2021.
+
The figure is constructed from <math>11</math> line segments, each of which has length <math>2</math>. The area of pentagon <math>ABCDE</math> can be written as <math>\sqrt{m} + \sqrt{n}</math>, where <math>m</math> and <math>n</math> are positive integers. What is <math>m + n ?</math>
 +
<asy> /* Made by samrocksnature */ pair A=(-2.4638,4.10658); pair B=(-4,2.6567453480756127); pair C=(-3.47132,0.6335248637894945); pair D=(-1.464483379039766,0.6335248637894945); pair E=(-0.956630463955801,2.6567453480756127); pair F=(-2,2); pair G=(-3,2); draw(A--B--C--D--E--A); draw(A--F--A--G); draw(B--F--C); draw(E--G--D); label("A",A,N); label("B",B,W); label("C",C,W); label("D",D,dir(0)); label("E",E,dir(0)); dot(A^^B^^C^^D^^E^^F^^G); </asy>
 +
<math>\textbf{(A)} ~20 \qquad\textbf{(B)} ~21 \qquad\textbf{(C)} ~22 \qquad\textbf{(D)} ~23 \qquad\textbf{(E)} ~24</math>
  
 
[[2021 AMC 12B Problems/Problem 15|Solution]]
 
[[2021 AMC 12B Problems/Problem 15|Solution]]
  
 
==Problem 16==
 
==Problem 16==
These problems will not be posted until the 2021 AMC 12B is released on Thursday, February 10, 2021.
+
Let <math>g(x)</math> be a polynomial with leading coefficient <math>1,</math> whose three roots are the reciprocals of the three roots of <math>f(x)=x^3+ax^2+bx+c,</math> where <math>1<a<b<c.</math> What is <math>g(1)</math> in terms of <math>a,b,</math> and <math>c?</math>
 +
 
 +
<math>\textbf{(A) }\frac{1+a+b+c}c \qquad \textbf{(B) }1+a+b+c \qquad \textbf{(C) }\frac{1+a+b+c}{c^2}\qquad \textbf{(D) }\frac{a+b+c}{c^2} \qquad \textbf{(E) }\frac{1+a+b+c}{a+b+c}</math>
  
 
[[2021 AMC 12B Problems/Problem 16|Solution]]
 
[[2021 AMC 12B Problems/Problem 16|Solution]]
  
 
==Problem 17==
 
==Problem 17==
These problems will not be posted until the 2021 AMC 12B is released on Thursday, February 10, 2021.
+
Let <math>ABCD</math> be an isosceles trapezoid having parallel bases <math>\overline{AB}</math> and <math>\overline{CD}</math> with <math>AB>CD.</math> Line segments from a point inside <math>ABCD</math> to the vertices divide the trapezoid into four triangles whose areas are <math>2, 3, 4,</math> and <math>5</math> starting with the triangle with base <math>\overline{CD}</math> and moving clockwise as shown in the diagram below. What is the ratio <math>\frac{AB}{CD}?</math>
 +
<asy>
 +
unitsize(100);
 +
pair A=(-1, 0), B=(1, 0), C=(0.3, 0.9), D=(-0.3, 0.9), P=(0.2, 0.5), E=(0.1, 0.75), F=(0.4, 0.5), G=(0.15, 0.2), H=(-0.3, 0.5);
 +
draw(A--B--C--D--cycle, black);
 +
draw(A--P, black);
 +
draw(B--P, black);
 +
draw(C--P, black);
 +
draw(D--P, black);
 +
label("$A$",A,(-1,0));
 +
label("$B$",B,(1,0));
 +
label("$C$",C,(1,-0));
 +
label("$D$",D,(-1,0));
 +
label("$2$",E,(0,0));
 +
label("$3$",F,(0,0));
 +
label("$4$",G,(0,0));
 +
label("$5$",H,(0,0));
 +
dot(A^^B^^C^^D^^P);
 +
</asy>
 +
<math>\textbf{(A)}\: 3\qquad\textbf{(B)}\: 2+\sqrt{2}\qquad\textbf{(C)}\: 1+\sqrt{6}\qquad\textbf{(D)}\: 2\sqrt{3}\qquad\textbf{(E)}\: 3\sqrt{2}</math>
  
 
[[2021 AMC 12B Problems/Problem 17|Solution]]
 
[[2021 AMC 12B Problems/Problem 17|Solution]]
  
 
==Problem 18==
 
==Problem 18==
These problems will not be posted until the 2021 AMC 12B is released on Thursday, February 10, 2021.
+
Let <math>z</math> be a complex number satisfying <math>12|z|^2=2|z+2|^2+|z^2+1|^2+31.</math> What is the value of <math>z+\frac 6z?</math>
 +
 
 +
<math>\textbf{(A) }-2 \qquad \textbf{(B) }-1 \qquad \textbf{(C) }\frac12\qquad \textbf{(D) }1 \qquad \textbf{(E) }4</math>
  
 
[[2021 AMC 12B Problems/Problem 18|Solution]]
 
[[2021 AMC 12B Problems/Problem 18|Solution]]
  
 
==Problem 19==
 
==Problem 19==
These problems will not be posted until the 2021 AMC 12B is released on Thursday, February 10, 2021.
+
Two fair dice, each with at least <math>6</math> faces are rolled. On each face of each die is printed a distinct integer from <math>1</math> to the number of faces on that die, inclusive. The probability of rolling a sum of <math>7</math> is <math>\frac34</math> of the probability of rolling a sum of <math>10,</math> and the probability of rolling a sum of <math>12</math> is <math>\frac{1}{12}</math>. What is the least possible number of faces on the two dice combined?
 +
 
 +
<math>\textbf{(A) }16 \qquad \textbf{(B) }17 \qquad \textbf{(C) }18\qquad \textbf{(D) }19 \qquad \textbf{(E) }20</math>
  
 
[[2021 AMC 12B Problems/Problem 19|Solution]]
 
[[2021 AMC 12B Problems/Problem 19|Solution]]
  
 
==Problem 20==
 
==Problem 20==
These problems will not be posted until the 2021 AMC 12B is released on Thursday, February 10, 2021.
+
Let <math>Q(z)</math> and <math>R(z)</math> be the unique polynomials such that<cmath>z^{2021}+1=(z^2+z+1)Q(z)+R(z)</cmath>and the degree of <math>R</math> is less than <math>2.</math> What is <math>R(z)?</math>
 +
 
 +
<math>\textbf{(A) }{-}z \qquad \textbf{(B) }{-}1 \qquad \textbf{(C) }2021\qquad \textbf{(D) }z+1 \qquad \textbf{(E) }2z+1</math>
  
 
[[2021 AMC 12B Problems/Problem 20|Solution]]
 
[[2021 AMC 12B Problems/Problem 20|Solution]]
  
 
==Problem 21==
 
==Problem 21==
These problems will not be posted until the 2021 AMC 12B is released on Thursday, February 10, 2021.
+
Let <math>S</math> be the sum of all positive real numbers <math>x</math> for which<cmath>x^{2^{\sqrt2}}=\sqrt2^{2^x}.</cmath>Which of the following statements is true?
 +
 
 +
<math>\textbf{(A) }S<\sqrt2 \qquad \textbf{(B) }S=\sqrt2 \qquad \textbf{(C) }\sqrt2<S<2\qquad \textbf{(D) }2\le S<6 \qquad \textbf{(E) }S\ge 6</math>
  
 
[[2021 AMC 12B Problems/Problem 21|Solution]]
 
[[2021 AMC 12B Problems/Problem 21|Solution]]
  
 
==Problem 22==
 
==Problem 22==
These problems will not be posted until the 2021 AMC 12B is released on Thursday, February 10, 2021.
+
Arjun and Beth play a game in which they take turns removing one brick or two adjacent bricks from one "wall" among a set of several walls of bricks, with gaps possibly creating new walls. The walls are one brick tall. For example, a set of walls of sizes <math>4</math> and <math>2</math> can be changed into any of the following by one move: <math>(3,2),(2,1,2),(4),(4,1),(2,2),</math> or <math>(1,1,2).</math>
 +
 
 +
<asy> unitsize(4mm); real[] boxes = {0,1,2,3,5,6,13,14,15,17,18,21,22,24,26,27,30,31,32,33}; for(real i:boxes){ draw(box((i,0),(i+1,3))); } draw((8,1.5)--(12,1.5),Arrow()); defaultpen(fontsize(20pt)); label(",",(20,0)); label(",",(29,0)); label(",...",(35.5,0)); </asy>
 +
 
 +
Arjun plays first, and the player who removes the last brick wins. For which starting configuration is there a strategy that guarantees a win for Beth?
 +
 
 +
<math>\textbf{(A) }(6,1,1) \qquad \textbf{(B) }(6,2,1) \qquad \textbf{(C) }(6,2,2)\qquad \textbf{(D) }(6,3,1) \qquad \textbf{(E) }(6,3,2)</math>
  
 
[[2021 AMC 12B Problems/Problem 22|Solution]]
 
[[2021 AMC 12B Problems/Problem 22|Solution]]
  
 
==Problem 23==
 
==Problem 23==
These problems will not be posted until the 2021 AMC 12B is released on Thursday, February 10, 2021.
+
Three balls are randomly and independently tossed into bins numbered with the positive integers so that for each ball, the probability that it is tossed into bin <math>i</math> is <math>2^{-i}</math> for <math>i=1,2,3,....</math> More than one ball is allowed in each bin. The probability that the balls end up evenly spaced in distinct bins is <math>\frac pq,</math> where <math>p</math> and <math>q</math> are relatively prime positive integers. (For example, the balls are evenly spaced if they are tossed into bins <math>3,17,</math> and <math>10.</math>) What is <math>p+q?</math>
 +
 
 +
<math>\textbf{(A) }55 \qquad \textbf{(B) }56 \qquad \textbf{(C) }57\qquad \textbf{(D) }58 \qquad \textbf{(E) }59</math>
  
 
[[2021 AMC 12B Problems/Problem 23|Solution]]
 
[[2021 AMC 12B Problems/Problem 23|Solution]]
  
 
==Problem 24==
 
==Problem 24==
These problems will not be posted until the 2021 AMC 12B is released on Thursday, February 10, 2021.
+
Let <math>ABCD</math> be a parallelogram with area <math>15</math>. Points <math>P</math> and <math>Q</math> are the projections of <math>A</math> and <math>C,</math> respectively, onto the line <math>BD;</math> and points <math>R</math> and <math>S</math> are the projections of <math>B</math> and <math>D,</math> respectively, onto the line <math>AC.</math> See the figure, which also shows the relative locations of these points.
 +
 
 +
<asy>
 +
size(350);
 +
defaultpen(linewidth(0.8)+fontsize(11));
 +
real theta = aTan(1.25/2);
 +
pair A = 2.5*dir(180+theta), B = (3.35,0), C = -A, D = -B, P = foot(A,B,D), Q = -P, R = foot(B,A,C), S = -R;
 +
draw(A--B--C--D--A^^B--D^^R--S^^rightanglemark(A,P,D,6)^^rightanglemark(C,Q,D,6));
 +
draw(B--R^^C--Q^^A--P^^D--S,linetype("4 4"));
 +
dot("$A$",A,dir(270));
 +
dot("$B$",B,E);
 +
dot("$C$",C,N);
 +
dot("$D$",D,W);
 +
dot("$P$",P,SE);
 +
dot("$Q$",Q,NE);
 +
dot("$R$",R,N);
 +
dot("$S$",S,dir(270));
 +
</asy>
 +
 
 +
Suppose <math>PQ=6</math> and <math>RS=8,</math> and let <math>d</math> denote the length of <math>\overline{BD},</math> the longer diagonal of <math>ABCD.</math> Then <math>d^2</math> can be written in the form <math>m+n\sqrt p,</math> where <math>m,n,</math> and <math>p</math> are positive integers and <math>p</math> is not divisible by the square of any prime. What is <math>m+n+p?</math>
 +
 
 +
<math>\textbf{(A) }81 \qquad \textbf{(B) }89 \qquad \textbf{(C) }97\qquad \textbf{(D) }105 \qquad \textbf{(E) }113</math>
  
 
[[2021 AMC 12B Problems/Problem 24|Solution]]
 
[[2021 AMC 12B Problems/Problem 24|Solution]]
  
 
==Problem 25==
 
==Problem 25==
These problems will not be posted until the 2021 AMC 12B is released on Thursday, February 10, 2021.
+
Let <math>S</math> be the set of lattice points in the coordinate plane, both of whose coordinates are integers between <math>1</math> and <math>30,</math> inclusive. Exactly <math>300</math> points in <math>S</math> lie on or below a line with equation <math>y=mx.</math> The possible values of <math>m</math> lie in an interval of length <math>\frac ab,</math> where <math>a</math> and <math>b</math> are relatively prime positive integers. What is <math>a+b?</math>
 +
 
 +
<math>\textbf{(A) }31 \qquad \textbf{(B) }47 \qquad \textbf{(C) }62\qquad \textbf{(D) }72 \qquad \textbf{(E) }85</math>
  
 
[[2021 AMC 12B Problems/Problem 25|Solution]]
 
[[2021 AMC 12B Problems/Problem 25|Solution]]
  
 
==See also==
 
==See also==
{{AMC12 box|year=2021|ab=B|before=[[2021 AMC 12A Problems]]|after=[[2022 AMC 12A Problems]]}}
+
{{AMC12 box|year=2021|ab=B|before=[[2021 AMC 12A Problems]]|after=[[2021 Fall AMC 12A Problems]]}}
  
 
[[Category:AMC 12 Problems]]
 
[[Category:AMC 12 Problems]]
 
{{MAA Notice}}
 
{{MAA Notice}}

Latest revision as of 12:31, 27 October 2023

2021 AMC 12B (Answer Key)
Printable versions: WikiAoPS ResourcesPDF

Instructions

  1. This is a 25-question, multiple choice test. Each question is followed by answers marked A, B, C, D and E. Only one of these is correct.
  2. You will receive 6 points for each correct answer, 2.5 points for each problem left unanswered if the year is before 2006, 1.5 points for each problem left unanswered if the year is after 2006, and 0 points for each incorrect answer.
  3. No aids are permitted other than scratch paper, graph paper, ruler, compass, protractor and erasers (and calculators that are accepted for use on the test if before 2006. No problems on the test will require the use of a calculator).
  4. Figures are not necessarily drawn to scale.
  5. You will have 75 minutes working time to complete the test.
1 2 3 4 5 6 7 8 9 10 11 12 13 14 15 16 17 18 19 20 21 22 23 24 25

Problem 1

How many integer values of $x$ satisfy $|x|<3\pi?$

$\textbf{(A) }9 \qquad \textbf{(B) }10 \qquad \textbf{(C) }18 \qquad \textbf{(D) }19 \qquad \textbf{(E) }20$

Solution

Problem 2

At a math contest, $57$ students are wearing blue shirts, and another $75$ students are wearing yellow shirts. The $132$ students are assigned into $66$ pairs. In exactly $23$ of these pairs, both students are wearing blue shirts. In how many pairs are both students wearing yellow shirts?

$\textbf{(A) }23 \qquad \textbf{(B) }32 \qquad \textbf{(C) }37 \qquad \textbf{(D) }41 \qquad \textbf{(E) }64$

Solution

Problem 3

Suppose\[2+\frac{1}{1+\frac{1}{2+\frac{2}{3+x}}}=\frac{144}{53}.\]What is the value of $x?$

$\textbf{(A) }\frac34 \qquad \textbf{(B) }\frac78 \qquad \textbf{(C) }\frac{14}{15} \qquad \textbf{(D) }\frac{37}{38} \qquad \textbf{(E) }\frac{52}{53}$

Solution

Problem 4

Ms. Blackwell gives an exam to two classes. The mean of the scores of the students in the morning class is $84$, and the afternoon class's mean score is $70$. The ratio of the number of students in the morning class to the number of students in the afternoon class is $\frac34$. What is the mean of the score of all the students?

$\textbf{(A) }74 \qquad \textbf{(B) }75 \qquad \textbf{(C) }76 \qquad \textbf{(D) }77 \qquad \textbf{(E) }78$

Solution

Problem 5

The point $P(a,b)$ in the $xy$-plane is first rotated counterclockwise by $90^\circ$ around the point $(1,5)$ and then reflected about the line $y=-x$. The image of $P$ after these two transformations is at $(-6,3)$. What is $b-a?$

$\textbf{(A) }1 \qquad \textbf{(B) }3 \qquad \textbf{(C) }5 \qquad \textbf{(D) }7 \qquad \textbf{(E) }9$

Solution

Problem 6

An inverted cone with base radius $12 \text{cm}$ and height $18\text{cm}$ is full of water. The water is poured into a tall cylinder whose horizontal base has a radius of $24\text{cm}$. What is the height in centimeters of the water in the cylinder?

$\textbf{(A) }1.5 \qquad \textbf{(B) }3 \qquad \textbf{(C) }4 \qquad \textbf{(D) }4.5 \qquad \textbf{(E) }6$

Solution

Problem 7

Let $N=34\cdot34\cdot63\cdot270.$ What is the ratio of the sum of the odd divisors of $N$ to the sum of the even divisors of $N?$

$\textbf{(A) }1:16 \qquad \textbf{(B) }1:15 \qquad \textbf{(C) }1:14 \qquad \textbf{(D) }1:8 \qquad \textbf{(E) }1:3$

Solution

Problem 8

Three equally spaced parallel lines intersect a circle, creating three chords of lengths $38,38,$ and $34$. What is the distance between two adjacent parallel lines?

$\textbf{(A) }5\frac12 \qquad \textbf{(B) }6 \qquad \textbf{(C) }6\frac12 \qquad \textbf{(D) }7 \qquad \textbf{(E) }7\frac12$

Solution

Problem 9

What is the value of\[\frac{\log_2 80}{\log_{40}2}-\frac{\log_2 160}{\log_{20}2}?\] $\textbf{(A) }0 \qquad \textbf{(B) }1 \qquad \textbf{(C) }\frac54 \qquad \textbf{(D) }2 \qquad \textbf{(E) }\log_2 5$

Solution

Problem 10

Two distinct numbers are selected from the set $\{1,2,3,4,\dots,36,37\}$ so that the sum of the remaining $35$ numbers is the product of these two numbers. What is the difference of these two numbers?

$\textbf{(A) }5 \qquad \textbf{(B) }7 \qquad \textbf{(C) }8\qquad \textbf{(D) }9 \qquad \textbf{(E) }10$

Solution

Problem 11

Triangle $ABC$ has $AB=13,BC=14$ and $AC=15$. Let $P$ be the point on $\overline{AC}$ such that $PC=10$. There are exactly two points $D$ and $E$ on line $BP$ such that quadrilaterals $ABCD$ and $ABCE$ are trapezoids. What is the distance $DE?$

$\textbf{(A) }\frac{42}5 \qquad \textbf{(B) }6\sqrt2 \qquad \textbf{(C) }\frac{84}5\qquad \textbf{(D) }12\sqrt2 \qquad \textbf{(E) }18$

Solution

Problem 12

Suppose that $S$ is a finite set of positive integers. If the greatest integer in $S$ is removed from $S$, then the average value (arithmetic mean) of the integers remaining is $32$. If the least integer in $S$ is also removed, then the average value of the integers remaining is $35$. If the greatest integer is then returned to the set, the average value of the integers rises to $40$. The greatest integer in the original set $S$ is $72$ greater than the least integer in $S$. What is the average value of all the integers in the set $S$?

$\textbf{(A) }36.2 \qquad \textbf{(B) }36.4 \qquad \textbf{(C) }36.6\qquad \textbf{(D) }36.8 \qquad \textbf{(E) }37$

Solution

Problem 13

How many values of $\theta$ in the interval $0<\theta\le 2\pi$ satisfy\[1-3\sin\theta+5\cos3\theta = 0?\] $\textbf{(A) }2 \qquad \textbf{(B) }4 \qquad \textbf{(C) }5\qquad \textbf{(D) }6 \qquad \textbf{(E) }8$

Solution

Problem 14

Let $ABCD$ be a rectangle and let $\overline{DM}$ be a segment perpendicular to the plane of $ABCD$. Suppose that $\overline{DM}$ has integer length, and the lengths of $\overline{MA},\overline{MC},$ and $\overline{MB}$ are consecutive odd positive integers (in this order). What is the volume of pyramid $MABCD?$

$\textbf{(A) }24\sqrt5 \qquad \textbf{(B) }60 \qquad \textbf{(C) }28\sqrt5\qquad \textbf{(D) }66 \qquad \textbf{(E) }8\sqrt{70}$

Solution

Problem 15

The figure is constructed from $11$ line segments, each of which has length $2$. The area of pentagon $ABCDE$ can be written as $\sqrt{m} + \sqrt{n}$, where $m$ and $n$ are positive integers. What is $m + n ?$ [asy] /* Made by samrocksnature */ pair A=(-2.4638,4.10658); pair B=(-4,2.6567453480756127); pair C=(-3.47132,0.6335248637894945); pair D=(-1.464483379039766,0.6335248637894945); pair E=(-0.956630463955801,2.6567453480756127); pair F=(-2,2); pair G=(-3,2); draw(A--B--C--D--E--A); draw(A--F--A--G); draw(B--F--C); draw(E--G--D); label("A",A,N); label("B",B,W); label("C",C,W); label("D",D,dir(0)); label("E",E,dir(0)); dot(A^^B^^C^^D^^E^^F^^G); [/asy] $\textbf{(A)} ~20 \qquad\textbf{(B)} ~21 \qquad\textbf{(C)} ~22 \qquad\textbf{(D)} ~23 \qquad\textbf{(E)} ~24$

Solution

Problem 16

Let $g(x)$ be a polynomial with leading coefficient $1,$ whose three roots are the reciprocals of the three roots of $f(x)=x^3+ax^2+bx+c,$ where $1<a<b<c.$ What is $g(1)$ in terms of $a,b,$ and $c?$

$\textbf{(A) }\frac{1+a+b+c}c \qquad \textbf{(B) }1+a+b+c \qquad \textbf{(C) }\frac{1+a+b+c}{c^2}\qquad \textbf{(D) }\frac{a+b+c}{c^2} \qquad \textbf{(E) }\frac{1+a+b+c}{a+b+c}$

Solution

Problem 17

Let $ABCD$ be an isosceles trapezoid having parallel bases $\overline{AB}$ and $\overline{CD}$ with $AB>CD.$ Line segments from a point inside $ABCD$ to the vertices divide the trapezoid into four triangles whose areas are $2, 3, 4,$ and $5$ starting with the triangle with base $\overline{CD}$ and moving clockwise as shown in the diagram below. What is the ratio $\frac{AB}{CD}?$ [asy] unitsize(100); pair A=(-1, 0), B=(1, 0), C=(0.3, 0.9), D=(-0.3, 0.9), P=(0.2, 0.5), E=(0.1, 0.75), F=(0.4, 0.5), G=(0.15, 0.2), H=(-0.3, 0.5);  draw(A--B--C--D--cycle, black);  draw(A--P, black); draw(B--P, black); draw(C--P, black); draw(D--P, black); label("$A$",A,(-1,0)); label("$B$",B,(1,0)); label("$C$",C,(1,-0)); label("$D$",D,(-1,0)); label("$2$",E,(0,0)); label("$3$",F,(0,0)); label("$4$",G,(0,0)); label("$5$",H,(0,0)); dot(A^^B^^C^^D^^P); [/asy] $\textbf{(A)}\: 3\qquad\textbf{(B)}\: 2+\sqrt{2}\qquad\textbf{(C)}\: 1+\sqrt{6}\qquad\textbf{(D)}\: 2\sqrt{3}\qquad\textbf{(E)}\: 3\sqrt{2}$

Solution

Problem 18

Let $z$ be a complex number satisfying $12|z|^2=2|z+2|^2+|z^2+1|^2+31.$ What is the value of $z+\frac 6z?$

$\textbf{(A) }-2 \qquad \textbf{(B) }-1 \qquad \textbf{(C) }\frac12\qquad \textbf{(D) }1 \qquad \textbf{(E) }4$

Solution

Problem 19

Two fair dice, each with at least $6$ faces are rolled. On each face of each die is printed a distinct integer from $1$ to the number of faces on that die, inclusive. The probability of rolling a sum of $7$ is $\frac34$ of the probability of rolling a sum of $10,$ and the probability of rolling a sum of $12$ is $\frac{1}{12}$. What is the least possible number of faces on the two dice combined?

$\textbf{(A) }16 \qquad \textbf{(B) }17 \qquad \textbf{(C) }18\qquad \textbf{(D) }19 \qquad \textbf{(E) }20$

Solution

Problem 20

Let $Q(z)$ and $R(z)$ be the unique polynomials such that\[z^{2021}+1=(z^2+z+1)Q(z)+R(z)\]and the degree of $R$ is less than $2.$ What is $R(z)?$

$\textbf{(A) }{-}z \qquad \textbf{(B) }{-}1 \qquad \textbf{(C) }2021\qquad \textbf{(D) }z+1 \qquad \textbf{(E) }2z+1$

Solution

Problem 21

Let $S$ be the sum of all positive real numbers $x$ for which\[x^{2^{\sqrt2}}=\sqrt2^{2^x}.\]Which of the following statements is true?

$\textbf{(A) }S<\sqrt2 \qquad \textbf{(B) }S=\sqrt2 \qquad \textbf{(C) }\sqrt2<S<2\qquad \textbf{(D) }2\le S<6 \qquad \textbf{(E) }S\ge 6$

Solution

Problem 22

Arjun and Beth play a game in which they take turns removing one brick or two adjacent bricks from one "wall" among a set of several walls of bricks, with gaps possibly creating new walls. The walls are one brick tall. For example, a set of walls of sizes $4$ and $2$ can be changed into any of the following by one move: $(3,2),(2,1,2),(4),(4,1),(2,2),$ or $(1,1,2).$

[asy] unitsize(4mm); real[] boxes = {0,1,2,3,5,6,13,14,15,17,18,21,22,24,26,27,30,31,32,33}; for(real i:boxes){ 	draw(box((i,0),(i+1,3))); } draw((8,1.5)--(12,1.5),Arrow()); defaultpen(fontsize(20pt)); label(",",(20,0)); label(",",(29,0)); label(",...",(35.5,0)); [/asy]

Arjun plays first, and the player who removes the last brick wins. For which starting configuration is there a strategy that guarantees a win for Beth?

$\textbf{(A) }(6,1,1) \qquad \textbf{(B) }(6,2,1) \qquad \textbf{(C) }(6,2,2)\qquad \textbf{(D) }(6,3,1) \qquad \textbf{(E) }(6,3,2)$

Solution

Problem 23

Three balls are randomly and independently tossed into bins numbered with the positive integers so that for each ball, the probability that it is tossed into bin $i$ is $2^{-i}$ for $i=1,2,3,....$ More than one ball is allowed in each bin. The probability that the balls end up evenly spaced in distinct bins is $\frac pq,$ where $p$ and $q$ are relatively prime positive integers. (For example, the balls are evenly spaced if they are tossed into bins $3,17,$ and $10.$) What is $p+q?$

$\textbf{(A) }55 \qquad \textbf{(B) }56 \qquad \textbf{(C) }57\qquad \textbf{(D) }58 \qquad \textbf{(E) }59$

Solution

Problem 24

Let $ABCD$ be a parallelogram with area $15$. Points $P$ and $Q$ are the projections of $A$ and $C,$ respectively, onto the line $BD;$ and points $R$ and $S$ are the projections of $B$ and $D,$ respectively, onto the line $AC.$ See the figure, which also shows the relative locations of these points.

[asy] size(350); defaultpen(linewidth(0.8)+fontsize(11)); real theta = aTan(1.25/2); pair A = 2.5*dir(180+theta), B = (3.35,0), C = -A, D = -B, P = foot(A,B,D), Q = -P, R = foot(B,A,C), S = -R; draw(A--B--C--D--A^^B--D^^R--S^^rightanglemark(A,P,D,6)^^rightanglemark(C,Q,D,6)); draw(B--R^^C--Q^^A--P^^D--S,linetype("4 4")); dot("$A$",A,dir(270)); dot("$B$",B,E); dot("$C$",C,N); dot("$D$",D,W); dot("$P$",P,SE); dot("$Q$",Q,NE); dot("$R$",R,N); dot("$S$",S,dir(270)); [/asy]

Suppose $PQ=6$ and $RS=8,$ and let $d$ denote the length of $\overline{BD},$ the longer diagonal of $ABCD.$ Then $d^2$ can be written in the form $m+n\sqrt p,$ where $m,n,$ and $p$ are positive integers and $p$ is not divisible by the square of any prime. What is $m+n+p?$

$\textbf{(A) }81 \qquad \textbf{(B) }89 \qquad \textbf{(C) }97\qquad \textbf{(D) }105 \qquad \textbf{(E) }113$

Solution

Problem 25

Let $S$ be the set of lattice points in the coordinate plane, both of whose coordinates are integers between $1$ and $30,$ inclusive. Exactly $300$ points in $S$ lie on or below a line with equation $y=mx.$ The possible values of $m$ lie in an interval of length $\frac ab,$ where $a$ and $b$ are relatively prime positive integers. What is $a+b?$

$\textbf{(A) }31 \qquad \textbf{(B) }47 \qquad \textbf{(C) }62\qquad \textbf{(D) }72 \qquad \textbf{(E) }85$

Solution

See also

2021 AMC 12B (ProblemsAnswer KeyResources)
Preceded by
2021 AMC 12A Problems
Followed by
2021 Fall AMC 12A Problems
1 2 3 4 5 6 7 8 9 10 11 12 13 14 15 16 17 18 19 20 21 22 23 24 25
All AMC 12 Problems and Solutions

The problems on this page are copyrighted by the Mathematical Association of America's American Mathematics Competitions. AMC logo.png